LSAT and Law School Admissions Forum

Get expert LSAT preparation and law school admissions advice from PowerScore Test Preparation.

 Administrator
PowerScore Staff
  • PowerScore Staff
  • Posts: 8917
  • Joined: Feb 02, 2011
|
#41348
Complete Question Explanation
(The complete setup for this game can be found here: lsat/viewtopic.php?t=13965)

The correct answer choice is (B)

If O is chosen for stop 2, then from the third rule O must be chosen for stop 4. Applying the first rule, O cannot be chosen for stop 5, and thus N must be chosen for stop 5. Because answer choices (A), (D), and (E) do not place N in stop 5, they can all be eliminated (answer choice (A) can also be eliminated because it violates the last rule).

In addition, if O is chosen for stop 2, then from the first rule O cannot be chosen for stop 1, and from the second rule O cannot be chosen for stop 6. This information eliminates answer choice (C), and thus answer choice (B) is correct.

Interestingly, every question in this game is a List question. This unusual occurrence is likely a result of the powerful interaction of the rules, and the fact that there are only two main directions the game can go. Most of the List questions in this game ask about partial lists, and this helps mask the true effects of the interaction of the rules.

Get the most out of your LSAT Prep Plus subscription.

Analyze and track your performance with our Testing and Analytics Package.